Sie sind auf Seite 1von 12

Chap 1 : Real Number www.rava.org.

in

CHAPTER 1
Real Number

Ans : [Board Term-1, 2015, Set-FHN8MGD]


TOPIC 1 : E D L
Here we can see easily that k is common factor
F T - between all and this is highest factor Thus HCF of
A k, 2k, 3k, 4k and 5k, is k .

7. Find the HCF and LCM of 90 and 144 by the method


of prime factorization.
VERY SHORT ANSWER TYPE QUESTIONS
Ans : [Board Term-1, 2012, Set-69]
1. Explain why 13233343563715 is a composite number?
We have 90 = 9 # 10
Ans : [Board Term-1, 2016 LGRKEGO]
= 2 # 32 # 5
The given number ends in 5. Hence it is a multiple of
5. Therefore it is a composite number. and 144 = 16 # 9
= 2 4 ×32
2. a and b are two positive integers such that the least
HCF = 2×32 = 18
prime factor of a is 3 and the least prime factor of b
is 5. Then calculate the least prime factor of (a + b). LCM = 2 4 ×32 ×5 = 720
Ans : [Board Term-1, 2014] NO NEED TO PURCHASE ANY BOOKS
a and b are two positive integers such that the least For session 2019-2020 free pdf will be available at
prime factor of a is 3 and the least prime factor of b www.cbse.online for
is 5. Then least prime factor of (a + b) is 2. 1. Previous 15 Years Exams Chapter-wise Question
Bank
3. What is the HCF of the smallest composite number
2. Previous Ten Years Exam Paper (Paper-wise).
and the smallest prime number?
3. 20 Model Paper (All Solved).
Ans : 4. NCERT Solutions
The smallest prime number is 2 and the smallest All material will be solved and free pdf. It will be
composite number is 4 = 22 . provided by 30 September and will be updated regularly.
Disclaimer : www.cbse.online is not affiliated to Central Board of Secondary Education,
Hence, required HCF (22, 2) = 2 . New Delhi in any manner. www.cbse.online is a private organization which provide free
study material pdfs to students. At www.cbse.online CBSE stands for Canny Books
4. Calculate the HCF of 33 ×5 and 32 ×52 . For School Education

Ans :
8. Using Euclid’s algorithm, find the HCF of 240 and
We have 33 ×5 = 32 ×5 # 3
288.
32 ×52 = 32 ×5 # 5 Ans : [Board Term-1, 2012, Set-35]
HCF (33 ×5, 32 ×52) = 32 ×5
We have 240 = 228×1 + 12
= 9×5 = 45
and 288 = 12×19 + 0
5. If HCF (a, b) = 12 and a×b = 1, 800 , then find LCM Hence, HCF of 240 and 228 = 12
(a, b).
Ans : 9. Given that HCF (306, 1314) = 18. Find LCM
(306, 1314)
We know that Ans : [Board Term-1, 2013, FFC]
HCF (a, b) # LCM (a, b) = a×b
We have HCF (306, 314) = 18
Substituting the values we have
LCM (306, 1314) = ?
12×LCM (a, b) = 1800
Let a = 306 and b = 1314 , then we have
1, 800
or, LCM (a, b) = = 150
12 LCM (a, b) # HCF (a, b) = a # b
Substituting values we have
SHORT ANSWER TYPE QUESTIONS - I or, LCM (a, b) ×18 = 306 # 1314

6. Find HCF of the numbers given below: or, LCM (a, b) = 306 # 1314
18
k, 2k, 3k, 4k and 5k, , where k is a Positive integer.
LCM (306, 1, 314) = 22, 338

Get all GUIDE and Sample Paper PDFs by whatsapp from +91 89056 29969 Page 1
Chap 1 : Real Number www.cbse.online
10. Complete the following factor tree and find the y = 1855 # 3 = 5565
composite number x . x = 2 # y = 2 # 5565 = 11130

Thus complete factor three is as given below.

Ans : [Board Term-1, 2015, Set - WJQZQBN

We have y = 5×13 = 65
and x = 3×195 = 585

11. Explain why (7×13×11) + 11 and


(7×6×5×4×3×2×1) + 3 are composite numbers.
Ans : [Board Term-1, 2012, Set-64] 13. Find the missing numbers a, b, c and d in the given
factor tree:
(7×13×11) + 11 = 11× (7×13 + 1)
= 11× (91 + 1)
= 11×92
and
(7×6×5×4×3×2×1) + 3 = 3 (7×6×5×4×2×1 + 1)
= 3× (1681) = 3 # 41 # 41
Since given numbers have more than two prime
factors, both number are composite.

12. Complete the following factor tree and find the


composite number x

Ans : [Board Term-1, 2012, Set-52]

We have a = 9009 = 3
3003

b = 1001 = 7
143

Since 143 = 11 # 13,


Thus c = 11 and d = 13 or c = 13 and d = 11
Ans : [Board Term-1, 2015, Set-DDE-M]
14. Complete the following factor tree and find the
We have z = 371 = 53
7

Download all GUIDE and Sample Paper pdfs from www.cbse.online or www.rava.org.in Page 2
Chap 1 : Real Number www.rava.org.in
composite number x . primes.
3×12×101 + 4 = 4 (3×3×101 + 1)
= 4 (909 + 1)
= 4 (910)
= 2 # 2 # (10 # 7 # 13)
= 2×2×2×5×7×13
= a composite number

NO NEED TO PURCHASE ANY BOOKS


For session 2019-2020 free pdf will be available at
www.cbse.online for
1. Previous 15 Years Exams Chapter-wise Question
Bank
2. Previous Ten Years Exam Paper (Paper-wise).
3. 20 Model Paper (All Solved).
4. NCERT Solutions
All material will be solved and free pdf. It will be
provided by 30 September and will be updated regularly.
Disclaimer : www.cbse.online is not affiliated to Central Board of Secondary Education,
New Delhi in any manner. www.cbse.online is a private organization which provide free
study material pdfs to students. At www.cbse.online CBSE stands for Canny Books
Ans : [Term-1, 2015, Set - 44, ], [Board term-2012, Set - 44] For School Education

We complete the given factor tree writing variable y


and z as following.
16. Complete the factor-tree and find the composite
number M .

We have z = 161 = 23
7

y = 7 # 161 = 1127
Composite number, x = 2 # 3381 = 6762

15. Explain whether 3×12×101 + 4 is a prime number or


a composite number.
Ans : [Board Term-1, 2016-17 Set; 193RQTQ, 2015, DDE-E]
A prime number (or a prime) is a natural number Ans : [NCERT]
greater than 1 that cannot be formed by multiplying
two smaller natural numbers. A natural number We have 91 = P # Q = 7 # 13
greater than 1 that is not prime is called a composite
So P = 7, Q = 13 or P = 13, Q = 7
number. For example, 5 is prime because the only ways
of writing it as a product, 1 # 5 or 5 # 1, involve O = 4095 = 3
1365
5 itself. However, 6 is composite because it is the
product of two numbers (2 # 3) that are both smaller N = 2×8190 = 16380
than 6. Every composite number can be written as
Composite number M = 16380×2 = 32760
the product of two or more (not necessarily distinct)

Get all GUIDE and Sample Paper PDFs by whatsapp from +91 89056 29969 Page 3
Chap 1 : Real Number www.cbse.online
Thus complete fact tree is shown below. a = bq + r
Take b = 4 , then 0 # r < 4 because 0 # r < b,
Thus a = 4q, 4q + 1, 4q + 2, 4q + 3
Here we can see easily that a = 4q, 4q + 2 are even, as
they are divisible by 2. Also 4q + 1, 4q + 3 are odd, as
they are not divisible by 2.
Thus any positive integer which has the form of
(4q + 1) or (4q + 3) is odd.

20. Can two numbers have 15 as their HCF and 175 as


their LCM? Give reasons.
Ans : [Board Term-1, 2012, Set-50]
LCM of two numbers should be exactly divisible by
their HCF. Since, 15 does not divide 175, two numbers
cannot have their HCF as 15 and LCM as 175.

21. Check whether 4n can end with the digit 0 for any
natural number n .
Ans : [Board Term-1, 2015, Set-FHN8MGD; NCERT]
If the number 4n, for any n, were to end with the digit
zero, then it would be divisible by 5 and 2.
That is, the prime factorization of 4n would contain
the prime 5 and 2. This is not possible because the
only prime in the factorization of 4n = 22n is 2. So, the
uniqueness of the Fundamental Theorem of Arithmetic
guarantees that there are no other primes in the
factorization of 4n . So, there is no natural number n
for which 4n ends with the digit zero. Hence 4n cannot
17. Find the smallest natural number by which 1200 end with the digit zero.
should be multiplied so that the square root of the For more files visit www.cbse.online

product is a rational number.


[Term-1, 2015, WJQZQBN, [Term-1, 2016, WV98HN3] 22. Show that 7n cannot end with the digit zero, for any
natural number n.
Ans : [Term-1, 2015, WJQZQBN, [Term-1, 2016, WV98HN3]
Ans : [Board Term-1, 2012, Set-63]
We have 1200 = 12 # 100 n
If the number 7 , for any n, were to end with the digit
= 4 # 3 # 4 # 25 zero, then it would be divisible by 5 and 2.
= 42 # 3 # 52 That is, the prime factorization of 7n would contain
Here if we multiply by 3, then its square root will be the prime 5 and 2. This is not possible because the
a rational number because all power will be 2. Thus only prime in the factorization of 7n = (1 # 7) n is 7.
the required smallest natural number is 3. 2 So, the uniqueness of the Fundamental Theorem of
Arithmetic guarantees that there are no other primes
18. Show that any positive even integer can be written in in the factorization of 7n . So, there is no natural
the form 6q, 6q + 2 or 6q + 4, where q is an integer. number n for which 7n ends with the digit zero. Hence
Ans : [Board Term1, 2016 Set ORDAWEZ] 7n cannot end with the digit zero.
Let a be any positive integer, then by Euclid’s division 23. Check whether (15) n can end with digit 0 for any
algorithm a can be written as n ! N.
a = bq + r Ans : [Board Term-1, 2012, Set-71]
Take b = 6 , then 0 # r < 6 because 0 # r < b, n
If the number (15) , for any n, were to end with the
Thus a = 6q, 6q + 1, 6q + 2, 6q + 3, 6q + 4, 6q + 5 digit zero, then it would be divisible by 5 and 2.
Here 6q, , 6q + 2 and 6q + 4 are divisible by 2 and so That is, the prime factorization of (15) n would contain
6q, 6q + 2 and 6q + 4 are even positive integers. the prime 5 and 2. This is not possible because the
only prime in the factorization of (15) n = (3 # 5) n are
Hence a is always an even integer if 3 and 5. The uniqueness of the Fundamental Theorem
a = 6q, 6q + 2, 6q + 4 of Arithmetic guarantees that there are no other
primes in the factorization of (15) n . Since there is no
19. Show that any positive odd integer is of the form prime factor 2, (15) n cannot end with the digit zero.
4q + 1 or 4q + 3, where q is some integer.
Ans : [Board Term-1, Set-70,55][NCERT] 24. The length, breadth and height of a room are 8 m
50 cm, 6 m 25 cm and 4 m 75 cm respectively. Find
Let a be any positive integer, then by Euclid’s division
the length of the longest rod that can measure the
algorithm a can be written as

Download all GUIDE and Sample Paper pdfs from www.cbse.online or www.rava.org.in Page 4
Chap 1 : Real Number www.rava.org.in
dimensions of the room exactly. Thus a = 5q, 5q + 1, 5q + 2, 5q + 3 and 5q + 4,
Ans : [Board Term-1, 2016 Set ORDAWEZ] Now a2 = (5q) 2 = 25q2 = 5 (5q2) = 5m
Here we have to determine the HCF of all length a2 = (5q + 1) 2 = 25q2 + 10q + 1 = 5m + 1
which can measure all dimension.
a2 = (5q + 2) 2 = 25q2 + 20q + 4 = 5m + 4
Length, l = 8m 50cm = 850cm cm Similarly a2 = (5q + 3) 2 = 5m + 4
2
= 50 # 17 = 2 # 5 # 17 and a2 = (5q + 4) 2 = 5m + 1
Breadth b = 6m 25 cm = 625 cm Thus square of any positive integer cannot be of the
= 25 # 25 = 52 # 52 form 5m + 2 or 5m + 3. 3
Height h = 4m 75cm = 475cm 28. Show that numbers 8n can never end with digit 0 of
= 25 # 19 = 52 # 19 any natural number n .
HCF(l, b, h) = HCF (850, 625, 475) Ans : [Board Term-1, 2015, Set-DDE-E][NCERT]

= 52 = 25 If the number 8n, for any n, were to end with the


digit zero, then it would be divisible by 5 and 2.
25. If two positive integers p and q are written as p = a2 b3 That is, the prime factorization of 8n would contain
and q = a3 b, where a and b are prime numbers than the prime 5 and 2. This is not possible because the
verify LCM (p, q) ×HCF (q, q) = p.q only prime in the factorization of (8) n = (23) n = 23n
Ans : [Sample Paper 2017] is 2. The uniqueness of the Fundamental Theorem of
Arithmetic guarantees that there are no other primes
We have p = a2 b3 = a # a # b # b # b in the factorization of (8) n . Since there is no prime
and q = a3 b = a # a # a # b factor 5, (8) n cannot end with the digit zero.
Now LCM (p, q) = a # a # a # b # b # b 29. Find the HCF, by Euclid’s division algorithm of the
=a b3 2 numbers 92690, 7378 and 7161.
and HCF (p, q) = a # a # b Ans : [Board Term-1, 2013, Set LK-59]

= a2 b By using Euclid’s Division Lemma, we have

LCM (p, q) ×HCF (p, q) = a3 b3 ×a2 b 92690 = 7378 # 12 + 4154

= a5 b 4 7378 = 4154×1 + 3224

= a2 b3 ×a3 b 4154 = 3224×1 + 930

= pq 3224 = 930×3 + 434

Add 8905629969 in Your Class Whatsapp Group to Get All PDFs 930 = 434×2 + 62
434 = 62×7 + 0
HCF (92690, 7378) = 62
SHORT ANSWER TYPE QUESTIONS - II Now, using Euclid’s Division Lemma on 7161 and 62,
we have
26. Find the HCF of 180, 252 and 324 by Euclid’s Division
7161 = 62×115 + 31
algorithm.
62 = 31×2 + 0
Ans : [Board Term-1, 2016 Set MV98HN3]
Thus HCF (7161, 62) = 31
We have 324 = 252×1 + 72
Hence, HCF of 92690, 7378 and 7161 is 31.
252 = 72×3 + 36
30. 144 cartons of Coke cans and 90 cartons of Pepsi cans
72 = 36×2 + 0
are to be stacked in a canteen. If each stack is of the
Thus HCF (324, 252) = 36 same height and if it equal contain cartons of the same
Now 180 = 36×5 + 0 drink, what would be the greatest number of cartons
Thus HCF (36, 180) = 36 each stack would have?
Ans : [Board Term-1, 2011, Set-66]
Thus HCF of 180, 252, and 324 is 36.
Hence required number = 999999 - 63 = 999936 1 The required answer will be HCF of 144 and 90.
144 = 2 4 ×32
27. Use Euclid division lemma to show that the square of
any positive integer cannot be of the form 5m + 2 or 90 = 2×32 ×5
5m + 3 for some integer m . HCF(144, 90) = 2×32 = 18
Ans : [Board Term-1, 2015, Set-FHN8MGD] Thus each stack would have 18 cartons.
Let a be any positive integer, then by Euclid’s division
31. Three bells toll at intervals of 9, 12, 15 minutes
algorithm a can be written as
respectively. If they start tolling together, after what
a = bq + r , 0 # r < b and q ! w time will they next toll together?
Take b = 5 , then 0 # r < 5 because 0 # r < b Ans : [Board Term-1, 2011, Set-44]

Get all GUIDE and Sample Paper PDFs by whatsapp from +91 89056 29969 Page 5
Chap 1 : Real Number www.cbse.online
The required answer is the LCM of 9, 12, and 15 m = 130 = 2
minutes. 6.5
Finding prime factor of given number we have, 35. Sho that any positive odd integer is of the form
6q + 1, 6q + 3 or 6q + 5, where q is some integer.
9 = 3×3 = 32
Ans : [Board Term-1, 2011, Set-60]
12 = 2×2×3 = 22 ×3
Let a be any positive integer, then by Euclid’s division
15 = 3 # 5 algorithm a can be written as
LCM(9, 12, 15) = 22 # 32 # 5 a = bq + r
= 150 minutes Take b = 6 , then 0 # r < 6 because 0 # r < b,
The bells will toll next together after 180 minutes. Thus a = 6q, 6q + 1, 6q + 2, 6q + 3, 6q + 4, 6q + 5
32. Find HCF and LCM of 16 and 36 by prime factorization Here 6q, , 6q + 2 and 6q + 4 are divisible by 2 and so
and check your answer. 6q, 6q + 2 and 6q + 4 are even positive integers.
Ans : But 6q + 1, 6q + 3, 6q + 5 are odd, as they are not
divisible by 2.
Finding prime factor of given number we have, Thus any positive odd integer is of the form
16 = 2×2×2×2 = 2 4 6q + 1, 6q + 3 or 6q + 5 .
36 = 2×2×3×3 = 22 ×32
36. Show that exactly one of the number n, n + 2 or n + 4
HCF(16, 36) = 2×2 = 4 is divisible by 3.
LCM (16, 36) = 2 ×34 2
Ans : [Sample Paper 2017]
= 16×9 = 144 If n is divisible by 3, clearly n + 2 andn + 4 is not
To check HCF and LCM by using formula divisible by 3.
If n is not divisible by 3, then two case arise as given
HCF(a, b) ×LCM(a, b) = a # b below.
or, 4×144 = 16×36 Case 1: n = 3k + 1
576 = 576 n + 2 = 3k + 1 + 2 = 3k + 3 = 3 (k + 1)
Thus LHS = RHS and n + 4 = 3k + 1 + 4 = 3k + 5 = 3 (k + 1) + 2
We can clearly see that in this case n + 2 is divisible
33. Find the HCF and LCM of 510 and 92 and verify that by 3 and n + 4 is not divisible by 3. Thus in this case
HCF × LCM = Product of two given numbers. only n + 2 is divisible by 3.
Ans : [Board Term-1, 2011, Set-39] Case 1: n = 3k + 2
Finding prime factor of given number we have, n + 2 = 3k + 2 + 2 = 3k + 4 = 3 (k + 1) + 1
92 = 22 ×23 and n + 4 = 3k + 2 + 4 = 3k + 6 = 3 (k + 2)
We can clearly see that in this case n + 4 is divisible
510 = 30 # 17 = 2×3×5×17 by 3 and n + 2 is not divisible by 3. Thus in this case
HCF (510, 92) = 2 only n + 4 is divisible by 3.
LCM (510, 92 = 22 ×23×3×5×14 Hence, exactly one of the numbers n, n + 2, n + 4, is
divisible by 3.
= 23460
HCF (510, 92) × LCM (510, 92)
= 2×23460 = 46920 LONG ANSWER TYPE QUESTIONS
Product of two numbers = 510×92 = 46920 37. Find HCF and LCM of 378, 180 and 420 by prime
Hence, HCF×LCM = Product of two numbers factorization method. Is HCF × LCM of these numbers
equal to the product of the given three numbers?
34. The HCF of 65 and 117 is expressible in the form Ans :
65m–117. Find the value of m . Also find the LCM of
65 and 117 using prime factorization method. Finding prime factor of given number we have,
Ans : [Board Term-1, 2011, Set-40] 378 = 2×33 ×7
Finding prime factor of given number we have, 180 = 22 ×32 ×5
117 = 13×2×3 420 = 22 ×3×7×5
65 = 13×5 HCF(378, 180, 420) = 2×3 = 6
HCF(117, 65) = 13 LCM(378, 180, 420) = 22 ×33 ×5×7
LCM(117, 65) = 13×5×3×3 = 585 = 22 ×33 ×5×7 = 3780
HCF = 65m - 117 HCF×LCM = 6×3780 = 22680
13 = 65m–117 Product of given numbers
65m = 117 + 13 = 130 = 378×180×420

Download all GUIDE and Sample Paper pdfs from www.cbse.online or www.rava.org.in Page 6
Chap 1 : Real Number www.rava.org.in
= 28576800 By using Euclid’s Division Lemma, we have
Hence, HCF × LCM ! Product of three numbers. 256 = 36×7 + 4

38. State Fundamental theorem of Arithmetic. Find LCM


36 = 4×9 + 0
of numbers 2520 and 10530 by prime factorization by Hence, the HCF of 256 and 36 is 4.
3. LCM : 256 = 28
Ans : [Board Term-1, 2016 Set-ORDAWEZ] 36 = 22 ×32
The fundamental theorem of arithmetic (FTA), also LCM (36, 256) = 28 ×32 = 256×9
called the unique factorization theorem or the unique-
prime-factorization theorem, states that every integer = 2304
greater than 1 either is prime itself or is the product HCF × LCM = Product of the two number
of a unique combination of prime numbers. 4×2, 304 = 256×36
OR 9216 = 9, 216
Every composite number can be expressed as the Hence verified.
product powers of primes and this factorization is
unique. 42. A fruit vendor has 990 apples and 945 oranges. He
Finding prime factor of given number we have, packs them into baskets. Each basket contains only
one of the two fruits but in equal number. Find the
2520 = 20 # 126 = 20 # 6 # 21 number of fruits to be put in each basket in order to
= 23 ×32 ×5×7 have minimum number of baskets.
10530 = 30 # 351 = 30 # 9 # 39 Ans : [Board Term-1, 2016 Set-O4YP6G7]
= 30 # 9 # 3 # 13 Required answer is the HCF of 990 and 945.
4 By using Euclid’s Division Lemma, we have
= 2×3 ×5×13
990 = 945 # 1 + 45
LCM(2520, 10530) = 23 ×3 4 ×5×7×13
945 = 45 # 21 + 0
= 294840
Thus HCF of 990 and 945 is 45. The fruit vendor
39. Can the number 6n , n being a natural number, end should put 45 fruits in each basket to have minimum
with the digit 5 ? Give reasons. number of baskets.
Ans : [Board Term-1, 2015, Set-WJQZBN] 43. For any positive integer n , prove that n3 - n is
n
If the number 6 for any n, were to end with the digit divisible by 6.
five, then it would be divisible by 5. Ans : [Board Term-1, 2015, 2012, Set-48]
That is, the prime factorization of 6n would contain
the prime 5. This is not possible because the only We have n3 - n = n (n2 - 1)
prime in the factorization of 6n = (2 # 3) n are 2 and = ^n - 1h n ^n + 1h
3. The uniqueness of the Fundamental Theorem of
Arithmetic guarantees that there are no other primes = ^n - 1h n ^n + 1h
Thus n3 - n is product of three consecutive positive
in the factorization of 6n . Since there is no prime
factor 5, 6n cannot end with the digit five. integers.
Since, any positive integers a is of the form 3q, 3q + 1
40. State Fundamental theorem of Arithmetic. Is it or 3q + 2 for some integer q .
possible that HCF and LCM of two numbers be 24 Let a, a + 1, a + 2 be any three consecutive integers.
and 540 respectively. Justify your answer.
Case I : a = 3q
Ans : [Board Term-1, 2015, Set-WJQZBN]
If a = 3q then,
Fundamental theorem of Arithmetic : Every integer
greater than one ither is prime itself or is the product a ^a + 1h^a + 2h = 3q ^3q + 1h^3q + 2h
of prime numbers and that this product is unique. Up Product of two consecutive integers ^3q + 1h and
to the order of the factors. ^3q + 2h is an even integer, say 2r .
LCM of two numbers should be exactly divisible by Thus a ^a + 1h^a + 2h = 3q ^2r h
their HCF. In other words LCM is always a multiple = 6qr , which is divisible by 6.
of HCF. Since, 24 does not divide 540 two numbers
cannot have their HCF as 24 and LCM as 540. Case II : a = 3q + 1

HCF = 24 If a = 3q + 1 then

LCM = 540 a ^a + 1h^a + 2h = ^3q + 1h^3q + 2h^3q + 3h


LCM = 540 = 22.5 not an integer = ^2r h^3 h^q + 1h
HCF 24 = 6r ^q + 1h which is divisible by
41. Find the HCF of 256 and 36 using Euclid’s Division 6.
Algorithm. Also, find their LCM and verify that HCF
× LCM = Product of the two numbers. Case III : a = 3q + 2
Ans : [Board Term-1, 2015, Set-DDE-E] If a = 3q + 2 then

Get all GUIDE and Sample Paper PDFs by whatsapp from +91 89056 29969 Page 7
Chap 1 : Real Number www.cbse.online
a ^a + 1h^a + 2h = ^3q + 2h^3q + 3h^3q + 4h = 81× ^- 38h + 237× ^13h
= 3 ^3q + 2h^q + 1h^3q + 4h = 81x + 237y ]
Here ^3q + 2h and = 3 ^3q + 2h^q + 1h^3q + 4h Hence x =- 38 and y = 13 . These values of x and y
are not unique.
= multiple of 6 every q
= 6r (say) 46. Show that the square of any positive integer is of the
which is divisible by 6. Hence, the product of three forms 4m or 4m + 1, where m is any integer.
consecutive integers is divisible by 6 and n3 - n is also Ans : [Board Term-1, 2012 Set-39]
divisible by 3. Let a be any positive integer, then by Euclid’s division
2 algorithm a can be written as
44. Prove that n - n is divisible by 2 for every positive
integer n . a = bq + r
Ans : [Board Term-1, 2012 Set-25] Take b = 4 , then 0 # r < 4 because 0 # r < b,

We have n2 - n = n ^n - 1h Thus a = 4q, 4q + 1, 4q + 2, 4q + 3


2
Thus n - n is product of two consecutive positive Case 1 : a = 4q
integers. a2 = ^4q h2 = 16q2 = 4 ^4q2h
Any positive integer is of the form 2q or 2q + 1, for
some integer q . = 4m
Case 1 : n = 2q where m = 4q2

If n = 2q we have Case 2 : a = 4q + 1

n ^n - 1h = 2q ^2q - 1h a2 = ^4q + 1h2 = 16q2 + 8q + 1


= 2m , = 4 ^4q2 + 2q h + 1
where m = q ^2q - 1h which is divisible by 2. = 4m + 1
Case 1 : n = 2q + 1 where m = 4q2 + 2q

If n = 2q + 1, we have Case 3 : a = 4q + 2

n ^n - 1h = (2q + 1) ^2q + 1 - 1h a2 = ^4q + 2h2


= 2q ^2q + 1h = 16q2 + 16q + 4
= 2m = 4 ^4q2 + 4q + 1h
where m = q ^2q + 1h which is divisible by 2. = 4m
Hence, n2 - n is divisible by 2 for every positive where m = 4q2 + 4q + 1
integer n . Case 4 : a2 = ^4q + 3h2 = 16q2 + 24q + 9
45. Find HCF of 81 and 237 and express it as a = 16q2 + 24q + 8 + 1
linear combination of 81 and 237 i.e. HCF = 4 ^4q2 + 6q + 2h + 1
^81, 237h = 81x + 237y for some x and y .
= 4m + 1
Ans : [Board Term-1, 2012 Set-35] [NCERT]
where m = 4q2 + 6q + 2
By using Euclid’s Division Lemma, we have From cases 1, 2, 3 and 4 we conclude that the square
237 = 81 # 2 + 75 ...(1) of any +ve integer is of the form 4m or 4m + 1.
81 = 75 # 1 + 6 ...(2)
47. Use Euclid’s Division Lemma to show that the cube
75 = 6 # 12 + 3 ...(3) of any positive integer is of the form 9m , 9m + 1, or
6 = 3#2+0 ...(4) 9m + 8, for some integer m .
Hence,HCF ^81, 237h = 3. Ans : [KVS, NCERT]

In order to write 3 in the form of 81x + 237y , Let a be any positive integer, then by Euclid’s division
algorithm a can be written as
3 = 75 - 6×12
a = bq + r
= 75 - ^81 - 75×1h ×12 Replace 6 from (2)
Take b = 3 , then 0 # r < 3 because 0 # r < b,
= 75 - 81×12 + 75×12
Thus a = 3q, 3q + 1, and 3q + 2
= 75 + 75×12 - 81×12
Case 1 : a = 3q
= 75 ^1 + 12h - 81×12
a3 = ^3q h3 = 27q3 = 9 ^3q3h
= 75×13 - 81×12
= 9m where m = 3q3
= 13 ^237 - 81×2h - 81×12 Replace 75 from (1)
Case 2 : a = 3q + 1
= 13×237 - 81×2×13 - 81×12
a3 = ^3q + 1h3
= 237×13 - 81 ^26 + 12h
= 27q3 + 9q ^3q + 1h + 1
= 237×13 - 81×38
= 9 ^3q3 + 3q2 + 1h + 1

Download all GUIDE and Sample Paper pdfs from www.cbse.online or www.rava.org.in Page 8
Chap 1 : Real Number www.rava.org.in
3 3 2
or a = 9m + 1 where m = 3q + 3q + 1 We have 6 = 6 = 6×23
1250 2×5 4 2×23 ×5 4
Case 3 : a = 3q + 2
a3 = ^3q + 2h3 3 3
= 64×2 4 = 6×24
= 27d3 + 18d ^3d + 2h + 8 2 ×5 (10)
= 9 ^3q3 + 6q2 + 4q h + 8 48 = 0.0048
=
or a3 = 9m + 8 where m = 3q2 + 6q2 + 4q 10000
Thus 6 will terminate after 4 decimal places.
From Case 1, 2 and 3, we conclude that the cube 1250
of any positive integer is of the form 9m, 9m + 1 or
9m + 8 for some integer m . 6. Write whether rational number 757 will have terminating
decimal expansion or a non-terminating decimal.
Ans : [Sample Paper 2017]
TOPIC 2 : I N , T -
7 = 7
N - - We have
75 3×52
, R D - Since denominator of given rational number is not
of form 2m ×5n , Hence, It is non-terminating decimal
expansion.

VERY SHORT ANSWER TYPE QUESTIONS SHORT ANSWER TYPE QUESTIONS - I


1. What is the condition for the decimal expansion of a 7. Show that 5 6 is an irrational number.
rational number to terminate? Explain with the help
Ans : [Board Term-1 2015, Set-CJEOQ]
of an example.
Ans : [Board Term-1, 2016 Set-O4YP6G7] Let 5 6 be a rational number, which can be expressed
as ab , where b ! 0; a and b are co-primes.
The decimal expansion of a rational number
terminates, if the denominator of rational number can Now 5 6 =a
b
be expressed as 2m 5n where m and n are non negative
integers and p and q both co-primes. 6 = a
3 = 3 = 0.3 5b
e.g.
10 21 ×51 or, 6 = rational
2. Find the smallest positive rational number by which But, 6 is an irrational number. Thus, our assumption
1
7 should be multiplied so that its decimal expansion is wrong. Hence, 5 6 is an irrational number. 1
terminates after 2 places of decimal.
Ans : [Board Term-1, 2016 Set LGRKEGO] 8. Write the denominator of the rational number 257 500 in
the form 2m ×5n, where m and n are non-negative
Since 1 × 7 = 1 = 0.01. integers. Hence write its decimal expansion without
7 100 100 actual division.
Thus smallest rational number is 7 Ans : [Board Term-1, 2012, Set-67, NCERT Exemplar]
100
3. What type of decimal expansion does a rational We have 500 = 25 # 20
number has? How can you distinguish it from decimal
expansion of irrational numbers? = 52 # 5 # 4
Ans : [Board Term-1, 2016 Set-ORDAWEZ] = 53 # 22
A rational number has its decimal expansion either Here denominator is 500 which can be written as
terminating or non-terminating, repeating An 22 ×53 .
irrational numbers has its decimal expansion non-
Now decimal expansion,
repeating and non-terminating.
257 = 257×2 = 514
500 2×22 ×53 103
4. Calculate 3 in the decimal form.
8 = 0.514
Ans :
9. Write a rational number between 2 and 3.
We have 3 = 3 = 2×53
8 23 2 3 ×5 3 Ans : [K.V.S.]
200 and 3 = 300
= 3753 = 375 We have 2=
100 100
10 1, 000
We need to find a rational number x such that
= 0.375
1 200 < x < 1 300
5. The decimal representation of 6
will terminate after 10 10
1250
how many places of decimal? Choosing any perfect square such as 225 or 256 in
Ans : between 200 and 300, we have

Get all GUIDE and Sample Paper PDFs by whatsapp from +91 89056 29969 Page 9
Chap 1 : Real Number www.cbse.online

x = 225 = 15 = 5 Hence, our assumption is false and 2 is irrational.


100 10 3
14. If p is prime number, then prove that p is an
Similarly if we choose 256, then we have irrational.
x = 256 = 16 = 8 Ans :
100 10 5
7 Let p be a prime number and if possible, let p be
10. Write the rational number will have a terminating
75
rational
decimal expansion. or a non-terminating repeating
decimal. Thus p =m,
n
Ans : [Sample Question Paper 2017,-18]
where m and n are co-primes and n ! 0 .
We have 7 = 7 Squaring on both sides, we get
75 3 # 52 2

The denominator of rational number 757 can not p = m2


n
be written in form 2m 5n So it is non-terminating
repeating decimal expansion. or, pn2 = m2 ...(1)
2 2
Here p divides pn . Thus p divides m and in result
p also divides m .
SHORT ANSWER TYPE QUESTIONS - II Let m = pq for some integer q and putting m = pq
11. Express ^ 154 + 405 h as a decimal fraction without actual in eq. (1), we have
division. pn2 = p2 q2
Ans : [Board Term-1, 2011, Set-74] or, n2 = pq2
15 + 5 = 15 × 25 + 5 × 25 Here p divides pq2 .Thus p divides n2 and in result p
We have
4 40 4 25 40 25 also divides n .
= 375 + 125
100 1000 [ a p is prime and p divides n2 & p divides n ]
Thus p is a common factor of m and n but this
= 3.75 + 0.125 = 3.875 contradicts the fact that m and n are primes. The
12. Express the number 0.3178 in the form of rational contradiction arises by assuming that p is rational.
Hence, p is irrational.
number a .
b
Ans : [Board Term-1, 2011, Set-A1][NCERT] 15. Prove that 3 + 5 is an irrational number.
Ans :
Let x = 0.3178
Assume that 3 + 5 is a rational number, then we
x = 0.3178178178
have
10, 000x = 3178.178178...
p
3+ 5 = , q!0
10x = 3.178178.... q
Subtracting, 9990x = 3175 p
5 = -3
q
or, x = 3175 = 635
9990 1998
p - 3q
13. Prove that 2 is an irrational number. 5 =
q
Ans : [Board Term-1, 2011, Set-A1. NCERT]
Here 5 is irrational and p -q 3q is rational. But rational
Let 2 be a rational number. number cannot be equal to an irrational number.
p Hence 3 + 5 is an irrational number.
Then 2 = ,
q
16. Prove that 5 is an irrational number and hence
where p and q are co-prime integers and q ! 0 On
show that 2 - 5 is also an irrational number.
squaring both the sides we have,
Ans : [Board Term-1, 2011, Set-60]
p2
2 = Assume that 5 be a rational number then we have
q2
or, p2 = 2p2 5 =a,
b
Since p2 is divisible by 2, thus p is also divisible by 2. (a ,b are co-primes and b ! 0 )
Let p = 2r for some positive integer r , then we have
a =b 5
p2 = 4r2
Squaring both the sides, we have
2q2 = 4r2
a2 = 5b2
or, q2 = 2r2
Thus 5 is a factor of a2 and in result 5 is also a factor
Since q2 is divisible by 2, thus q is also divisible by 2.
We have seen that p and q are divisible by 2, which of a .
contradicts the fact that p and q are co-primes. Let a = 5c where c is some integer, then we have
Download all GUIDE and Sample Paper pdfs from www.cbse.online or www.rava.org.in Page 10
Chap 1 : Real Number www.rava.org.in
2 2 2 2
a = 25c b = 3c
2 2
Substituting a = 5b we have Thus 3 is a factor of b2 and in result 3 is also a factor
5b2 = 25c2 of b .
2
b = 5c 2 Thus 3 is a common factor of a and b . But this
contradicts the fact that a and b are co-primes. Thus,
Thus 5 is a factor of b2 and in result 5 is also a factor our assumption that 3 is rational number is wrong.
of b . Hence 3 is irrational.
Thus 5 is a common factor of a and b . But this
Let us assume that 7 + 2 3 be rational equal to a ,
contradicts the fact that a and b are co-primes. Thus,
our assumption that 5 is rational number is wrong. then we have
Hence 5 is irrational. p
7+2 3 =
q
Let us assume that 2 - 5 be rational equal to a , q ! 0 and p and q are co-primes
then we have
2- 5 =a p p - 7q
2 3 = -7 =
q q
2-a = 5
Since we have assume 2 - a is rational, but 5 is p - 7q
or 3 =
not rational. Rational number cannot be equal to an 2q
irrational number. Thus 2 - 5 is irrational. Here p - 7q and 2q both are integers, hence 3
should be a rational number. But this contradicts
17. If two positive integers p and q are written as p = a2 b3 the fact that 3 is an irrational number. Hence our
and q = a3 b , a and b are prime number then. Verify. assumption is not correct and 7 + 2 3 is irrational.
LCM × ^p.q.h × HCF ^p.q.h = pq.
19. Show that there is no positive integer n , for which
Ans : [Sample Question Paper 2017-18]
n - 1 + n - 1 is rational.
2 3
We have p = a b = a#a#b#b#b Ans : [Board Term-1, 2012, Set-48]
and q = a3 b = a # a # a # b Let us assume that there is a positive integer n for
Now LCM (p, q) = a # a # a # b # b # b which n - 1 + n - 1 is rational and equal to qp ,
where p and q are positive integers and ^q ! 0h .
= a3 b2
p
and HCF (p, q) = a # a # b n-1+ n-1 = ...(1)
q
= a2 b
q 1
LCM (p, q) ×HCF (p, q) = a3 b3 ×a2 b or, =
p n-1+ n+1
= a5 b 4
= n-1- n+1
= a2 b3 ×a3 b ^ n - 1 + n + 1 h^ n - 1 - n + 1 h
= pq n-1- n+1
=
(n - 1) - (n + 1)

LONG ANSWER TYPE QUESTIONS q n-1- n+1


or =
p -2
18. Prove that 3 is an irrational number. Hence, show
2q
that 7 + 2 3 is also an irrational number. n+1- n-1 = ...(2)
p
Ans : [Board Term-1, 2012, Set-DDE-M] Adding (1) and (2), we get
Assume that 3 be a rational number then we have p 2q p2 + 2q2
2 n+1 = + =
q p pq
3 =a, ...(3)
b
(a ,b are co-primes and b ! 0 )
Subtracting (2) from (1) we have

a =b 3 p2 - 2q2
2 n-1 =
pq
Squaring both the sides, we have ...(4)
a2 = 3b2
From (3) and (4), we observe that n + 1 and n - 1
Thus 3 is a factor of a2 and in result 3 is also a factor both are rational because p and q both are rational.
of a . But it possible only when ^n + 1h and ^n - 1h both are
Let a = 3c where c is some integer, then we have perfect squares. But they differ by 2 and two perfect
squares never differ by 2. So both ^n + 1h and ^n - 1h
a2 = 9c2 cannot be perfect squares, hence there is no positive
Substituting a2 = 9b2 we have integer n for which n - 1 + n + 1 is rational.
3b2 = 9c2

Get all GUIDE and Sample Paper PDFs by whatsapp from +91 89056 29969 Page 11
Chap 1 : Real Number www.cbse.online
HOTS QUESTIONS 657 = 306×2 + 45

20. Show that 571 is a prime number.


306 = 45×6 + 36
Ans : 45 = 36×1 + 9
36 = 9 # 4 + 0
Let x = 571
HCF ^657, 963h = 9
x = 571
Now 571 lies between the perfect squares of ^23h2 = 529 Now 9 = 657x + 963× ^- 15h
and ^24h2 = 576 . Prime numbers less than 24 are 2, 3, or, 657x = 9 + 963×15
5, 7, 11, 13, 17, 19, 23. Here 571 is not divisible by any = 9 + 14445
of the above numbers, thus 571 is a prime number.
or, 657x = 14454
21. Find the least number that is divisible by all numbers
or, x = 14454 = 22
between 1 and 10 (both inclusive). 657
Ans : 25. Express the HCF/LCM of 48 and 18 as a linear
The required number is the LCM of 1, 2, 3, 4, 5, 6, 7, combination.
8, 9, 10, Ans :
LCM = 2 # 2 # 3 # 2 # 3 # 5 # 7 Using Euclid’s Division Lemma, we have
= 2520 48 = 18×2 + 12 (1)
18 = 12×1 + 6 (2)
22. An army contingent of 104 members is to march
behind an army band of 96 members in a parade. 12 = 6×2 + 0
The two groups are to march in the same number of Thus HCF ^18, 48h = 6
columns in which they can march ?
Now 6 = 18 - 12×1
Ans : [Board Term-1, 2012, Set-52]
From (2)
Let the number of columns be x which is the largest
number, which should divide both 104 and 96. It 6 = 18 - (48 - 18×2)
means x should be HCF of 104 and 96. From (1)
By using Euclid’s Division Lemma, we have 6 = 18 - 48×1 + 18×2
104 = 96 # 1 + 8 6 = 18 # (2 + 1) - 48 # 1
96 = 8 # 12 + 0 6 = 18×3 - 48×1
Thus HCF of 104 and 96 is 8 and columns are required. 6 = 18×3 + 48× ^- 1h
23. If d is the HCF of 30 and 72, find the value of x and Thus 6 = 18x + 48y ,
y satisfying d = 30x + 72y . where x = 3, y =- 1
Ans : Here x and y are not unique.
Using Euclid’s Division Lemma, we have 6 = 18×3 + 48× ^- 1h
72 = 30 # 2 + 12 ...(1) = 18 # 3 + 48 # ^- 1h + 18 # 48 - 18 # 48
30 = 12 # 2 + 6 ...(2) = 18 (3 + 48) + 48 ^- 1 - 18h
12 = 6 # 2 + 0 ...(3) = 18×51 + 48× ^- 19h
Thus HCF(30, 72) = 6 6 = 18x + 48y ,
Now 6 = 30 - 12 # 2 where x = 51, y = - 19
From (2) For more files visit www.cbse.online
6 = 30 - (72 - 30 # 2) # 2
NO NEED TO PURCHASE ANY BOOKS
From (1)
For session 2019-2020 free pdf will be available at
6 = 30 - 72 # 2 + 30 # 4 www.cbse.online for
6 = 30 (1 - 4) - 72 # 2 1. Previous 15 Years Exams Chapter-wise Question
6 = 30 # 5 + 72 # (- 2) Bank
2. Previous Ten Years Exam Paper (Paper-wise).
6 = 30x + 72y 3. 20 Model Paper (All Solved).
Thus x = 5 and y =- 2 .Here x and y are not unique. 4. NCERT Solutions
All material will be solved and free pdf. It will be
24. If HCF of 657 and 963 is expressible in the form of provided by 30 September and will be updated regularly.
657x + 963 # (- 15), find the value of x . Disclaimer : www.cbse.online is not affiliated to Central Board of Secondary Education,
New Delhi in any manner. www.cbse.online is a private organization which provide free
Ans : study material pdfs to students. At www.cbse.online CBSE stands for Canny Books
For School Education
Using Euclid’s Division Lemma we have
963 = 657×1 + 306

Download all GUIDE and Sample Paper pdfs from www.cbse.online or www.rava.org.in Page 12

Das könnte Ihnen auch gefallen